Đến nội dung

Trung Gauss

Trung Gauss

Đăng ký: 01-04-2014
Offline Đăng nhập: 10-04-2018 - 19:02
*****

#560680 ĐỀ KIỂM TRA ĐỘI TUYỂN HSG TRƯỜNG CHUYÊN LONG AN

Gửi bởi Trung Gauss trong 21-05-2015 - 11:51

$f(0)=0$
thay $y=0$ vào (*) ta có $f(x^2)=x^2$, suy ra $f(x)=x$, với mọi $x \ge 0$
thay $x=0$ vào (*) ta có $f(0)=f(y)f(-y)+y^2$

suy ra $f(y)f(-y)=-y^2, \forall y \Longrightarrow xf(-x)=-x^2, \forall x>0$

$\Longrightarrow f(-x)=-x, \forall x>0$

kết hợp $f(0)=0$ suy ra $f(x)=x, \forall x$

Vậy chỉ có một hàm thỏa bài toán thôi :))

 

Oh thử nhầm  :D




#560657 ĐỀ KIỂM TRA ĐỘI TUYỂN HSG TRƯỜNG CHUYÊN LONG AN

Gửi bởi Trung Gauss trong 21-05-2015 - 10:24

 

ĐỀ KIỂM TRA ĐỘI TUYỂN HSG

Năm học: 2014-2015

Thời gian: 150 phút

 

 

Câu 5 (2 điểm): 

Tìm tất cả các hàm số $f:\mathbb{R}\to\mathbb{R}$ thỏa: $$f(x^2)=f(y).f(2x-y)+(y-x)^2,\forall x,y\in\mathbb{R}$$

 

 

 

Lời giải:

 Trong $(*)$ cho $x=y$ ta được: $$f(x^2)=f^2(x),\forall x\in\mathbb{R}\;\;\;\;\;\;\;\;\;\;\;\;\;\;\;\;(1)$$ Trong $(1)$ lấy $x=0$ ta được $f(0)=0\vee f(0)=1$.

Trường hợp 1: $f(0)=0$, trong $(*)$ cho $y=0$ ta được: $$\begin{aligned}&\;\;\;\;f(x^2)=x^2\\&\Leftrightarrow f^2(x)=x^2, \forall x\in\mathbb{R}\end{aligned}$$ Từ đây suy ra $f(x)=x$ hoặc $f(x)=-x$. Thử lại ta thấy chỉ có hàm số $f(x)=x,\forall x\in\mathbb{R}$ thỏa mãn bài toán.

Trường hợp 2: $f(0)=1$, trong $(*)$ cho $y=0$, ta được: $$f(x^2)=f(2x)+x^2,\forall x\in\mathbb{R}\;\;\;\;\;\;\;\;\;\;\;\;\;\;\;(2)$$ Trong $(*)$ thay $y$ bởi $x-y$ ta được: $$f(x^2)=f(x-y)f(x+y)+y^2,\forall x,y\in\mathbb{R}\;\;\;\;\;\;\;\;\;\;\;\;\;(3)$$ Kết hợp $(2)$ và $(3)$ ta được: $$f(2x)+x^2=f(x+y).f(x-y)+y^2,\forall x, y\in\mathbb{R}$$ Đặt $a=x+y,\;b=x-y, \;a,b\in\mathbb{R}$ ta được: $$f(a+b)+ab=f(a).f(b),\forall a,b\in\mathbb{R}\;\;\;\;\;\;\;\;\;\;\;\;\;\;\;\;\;\;(4)$$ Trong $(*)$, ta thay $y$ bởi $b$, $2x-y$ bởi $a$, ta được: $$f\left(\dfrac{(a+b)^2}{4}\right)=f(a).f(b)-\dfrac{(a-b)^2}{4},\forall a, b\in\mathbb{R}\;\;\;\;\;\;\;\;\;\;\;\;\;\;\;\;\;(5)$$ Kết hợp $(4)$ và $(5)$ ta được:  $$\begin{aligned}&\;\;\;f(a+b)+ab=f\left(\dfrac{(a+b)^2}{4}\right)-\dfrac{(a-b)^2}{4},\forall a, b\in\mathbb{R}\\&\Leftrightarrow 4f(a+b)=4f\left(\dfrac{(a+b)^2}{4}\right)-(a+b)^2,\forall a,b\in\mathbb{R}\;\;\;\;\;\;\;\;\;(6)\end{aligned}$$ Trong $(6)$ cho $a+b=4$ ta được $-4^2=0$, vô lý.

Vậy chỉ có hàm số thỏa mãn bài toán: $$f(x)=x,\forall x\in\mathbb{R}$$




#560512 ĐỀ KIỂM TRA ĐỘI TUYỂN HSG TRƯỜNG CHUYÊN LONG AN

Gửi bởi Trung Gauss trong 20-05-2015 - 16:00

ĐỀ KIỂM TRA ĐỘI TUYỂN HSG

Năm học: 2014-2015

Thời gian: 150 phút

 

I. Phần chung (Cho tất cả thí sinh):

Câu 1 (4 điểm):

      a. Cho $x, y, z$ là các số thực không âm thỏa $x+y+z=2$. CMR: $$x^3+y^3+z^3\le 1+\dfrac{1}{2}(x^4+y^4+z^4)$$

      b. Xét số thực $x,y$ thỏa mãn điều kiện $36x^2+16y^2=9$. Tìm GTLN và GTNN của $P=y-2x+5$

Câu 2 (4 điểm):

 Giải phương trình: $$x^2+\sqrt[3]{x^4-x^2}=2x+1$$ 

Câu 3 ( điểm):

 Giả sử $D$ là một điểm trên cạnh $BC$ ($D$ khác $B, C$) của tam giác $ABC$. Gọi $E, F$ lần lượt là tâm đường tròn nội tiếp tam giác $ABD, ACD$. CMR nếu $B, C, D, F$ đồng viên thì $\dfrac{AD+BD}{AD+DC}=\dfrac{AB}{AC}$

Câu 4 (2 điểm):

  Cho $p$ là một số nguyên tố lẻ và $a, b$ là hai số tự nhiên sao cho $a+b$ chia hết cho $p$ và $a-b$ chia hết cho $p-1$. CMR $a^b+b^a$ chia hết cho $2p$

Câu 5 (2 điểm): 

Tìm tất cả các hàm số $f:\mathbb{R}\to\mathbb{R}$ thỏa: $$f(x^2)=f(y).f(2x-y)+(y-x)^2,\forall x,y\in\mathbb{R}$$

Câu 6 ( điểm): 

Đề thi HKII môn Vật lý có $50$ câu trắc nghiệm, mỗi câu có $4$ phương án. Trong đó có một phương án đúng, trả lời đúng mỗi câu được $0,2$ điểm. Một thí sinh làm được $40$ câu trong đó có $32$ câu đúng. Ở $10$ câu còn lại thí sinh chọn ngẫu nhiên $1$ trong $4$ phương án. Tính xác suất để thí sinh đạt $8$ điểm trở lên.

II. Phần tự chọn (HS chọn câu 7a hoặc câu 7b):

Câu 7a: (2 điểm)

  Cho $a, c>0$. Xét dãy số: $$\begin{cases}x_1=a\\x_{n+1}=c x^2_{n}+x_n,\forall n\in\mathbb{N^*}\end{cases}$$. CMR:$x_n\ge \sqrt{c^{n-1}.n^n.a^{n+1}},\forall n\in\mathbb{N^*}$.

Câu 7b: (2 điểm)

 Do sản xuất bị lỗi, một mảnh vải hình vuông cạnh $1,2$m có $31$ lỗ thủng nhỏ như kim châm. Có thể lợi dụng mảnh vải này để cắt ra một khăn nhỏ hình tròn bán kính $0,1$m không chứa lỗ thủng nào cả hay không?

 

Nguồn: Đề được lấy từ FB của bạn Phạm Quốc Thắng- Hades Phạm




#551416 Đề thi Olympic 30/4 chính thức lần thứ XXI năm 2015 (lớp 11)

Gửi bởi Trung Gauss trong 04-04-2015 - 19:57

 

KỲ THI OLYMPIC 30/4 LẦN THỨ XXI 

 

 

Bài 5 (3 điểm) 

Tìm tất cả các hàm số đơn ánh $f:\mathbb{R}\rightarrow \mathbb{R}$ và thoả :

$$f(x^3)+f(y^3)=(x+y)\left [ f^2(x)-f(x)f(y)+f^2(y) \right ],\;\forall x,y\in \mathbb{R}\;\;\;\;\;\;\;\;\;\;\;\;\;\;\;\;\;\;\;(1)$$

 

 

 

 

Lời giải: 

Trong $(1)$ cho $x=y$ ta được: $$f(x^3)=xf^2(x),\;\forall x\in\mathbb{R}\;\;\;\;\;\;\;\;\;\;\;\;\;\;\;\;\;\;(2)$$ Trong $(2)$ cho $x=0$, ta được: $f(0)=0$

Trong $(2)$ cho $x=1$ ta được $f(1)=0\;\vee f(1)=1$.

 

Trường hợp 1: $f(1)=0$. Trong $(1)$ cho $y=1$ ta được: $$f(x^3)=(x+1)f^2(x),\forall x\in\mathbb{R}\;\;\;\;\;\;\;\;\;\;\;\;\;\;\;\;(3)$$ Kết hợp với: $$f(x^3)=xf^2(x),\forall x\in\mathbb{R}\;\;\;\;\;\;\;\;\;\;\;\;\;\;\;\;\;(4)$$ Từ $(3)$ và $(4)$ suy ra: $f(x0\equiv 0, \forall x\in\mathbb{R}$ Tuy nhiên vì $f$ đơn ánh nên ta loại nghiệm này.

 

Trường hợp 2: $f(1)=1$. Trong $(1)$ cho $y=1$ ta được: $$f(x^3)+1=(x+1)\left[ f^2(x)-f(x)+1\right],\;\forall x\in\mathbb{R}$$ Kết hợp với $$f(x^3)=xf^2(x),\forall x\in\mathbb{R}$$ Suy ra: $$\begin{aligned}&\;\;\;\;\;\;  xf^2(x)+1=(x+1)\left[ f^2(x)-f(x)+1\right]\\& \Leftrightarrow f^2(x)-xf(x)-f(x)+x=0\\& \Leftrightarrow \left[ f(x)-1\right]\left[f(x)-x\right] =0\end{aligned}$$ Tuy nhiên vì $f$ đơn ánh nên $f(x)=1\Leftrightarrow x=1$, tức là ta thu được $f(x)=x,\forall x\in\mathbb{R}$. Kết luận: $$ f(x)\equiv x.\;\forall x\in\mathbb{R}$$




#545167 Đề thi chính thức Olympic 30-4 toán 11 lần thứ XX năm 2014

Gửi bởi Trung Gauss trong 21-02-2015 - 19:16

 

ĐỀ THI OLYMPIC 30/4 TRUYỀN THỐNG LỚP 11 LẦN XX NĂM 2014

 

Câu 4: (3 điểm)
Xác định các đa thức $P(x)$ hệ số thực thỏa mãn $P(x)P(x^2)=P(x^3+3x), \forall x\in \mathbb{R}$
 

 

 


Lời giải:

Dễ thấy nếu $P(x)$ là đa thức hằng thì có hai đa thức thỏa mãn bài toán là $P(x)\equiv 0,\forall x\in\mathbb{R}$ và $P(x)\equiv 1,\forall x\in\mathbb{R}$. Ta xét trong trường hợp $P(x)$ khác hằng.

Trong $(*)$ cho $x=0$, ta được $P(0)=0$ hoặc $P(0)=1$.


    Trường hợp 1: $P(0)=0$. Đặt $P(x)=x^k.Q(x)$, trong đó $k\ge 1, Q(0)\neq 0$, thay vào $(*)$ ta được: $$x^{3k}Q(x).Q(x^2)=(x^3+3x)^k.Q(x^3+3x),\forall x\in\mathbb{R}\\\Leftrightarrow x^{2k}.Q(x).Q(x^2)=(x^2+3)^k.Q(x^3+3x),\forall x\in\mathbb{R^*}$$ Do $Q(0)\neq 0$ nên $Q(x)$ có hệ số tự do khác $0$. Gọi hệ số này là $d$. Ta nhận thấy:

       $\bullet$ Hệ số bậc nhỏ nhất của vế trái là: $d^2$

       $\bullet$ Hệ số bậc nhỏ nhất của vế phải là: $3^kd$

 Từ đó suy ra: $$d^2=3^k d\Leftrightarrow d=0, \text{ hay } Q(0)=0,\text{vô lý}$$ Do vậy, ta thu được $P(x)\equiv 0, \forall x\in\mathbb{R}$, thử lại thấy thỏa $(*)$.


    Trường hợp 2: $P(0)=1$. Đặt $P(x)=x^h.F(x)+1$ trong đó:  $h\ge 1, F(0)\neq 0$, thay vào $(*)$ ta được: $$x^{3h}F(x).F(x^2)+x^hF(x)+x^{2h}F(x^2)=(x^3+3x)^hF(x^3+3x),\forall x\in\mathbb{R}\\\Leftrightarrow x^{2h} F(x).F(x^2)+F(x)+x^hF(x^2)=(x^2+3)^h.F(x^3+3x),\forall x\neq 0$$ Do $F(0)\neq 0$ nên tồn tại $l\neq 0$ là hệ số tự do của $F(x)$. Tương tự như trường hợp 1, ta nhận thấy:

       $\bullet$ Hệ số bậc nhỏ nhất của vế trái là $l$

       $\bullet$ Hệ số bậc nhỏ nhất của vế phải là $3^h l$

  Từ đó suy ra: $$l=3^h.l\Leftrightarrow l=0,\text{ vô lý}$$

 Như vậy, ta thu được $P(x)\equiv 1, \forall x\in\mathbb{R}$, thử lại thấy thỏa $(*)$.

 Tóm lại, có hai đa thức thỏa mãn bài toán $$\boxed{P(x)\equiv 0,\forall x\in\mathbb{R}\\\\P(x)\equiv 1,\forall x\in\mathbb{R}}$$




#543668 Tổng hợp các định lý trong hình học và ứng dụng của chúng qua các bài toán

Gửi bởi Trung Gauss trong 10-02-2015 - 19:17

 Định lý 4:

   ĐỊNH LÝ CASEY: Cho bốn đường tròn $C_i,\,i=\overline{1,4}$. Kí hiệu $t_{ij}$ là độ dài của tiếp tuyến hai đường tròn $C_i$ và $C_j$. Khi đó bốn đường tròn $C_i$ cùng tiếp xúc với một đường thẳng hoặc đường tròn $C$ khi và chỉ khi: $$t_{12}t_{34}\;^{+}_{-}t_{13}t_{42}\;^{+}_{-}t_{14}t_{23}=0$$
 
Chứng minh: 
   Bổ đề: Cho hai đường tròn $C_1 (O_1, r_1)$ và $(C_2()_2, r_2)$ cùng tiếp xúc với đường tròn $C(O, R)$ tại $A, B$ tương ứng. Khi đó độ dài tiếp tuyến chung trong hoặc ngoài của $C_1$ và $C_2$ xác định bởi: $$t_{12}=\dfrac{AB}{R}\sqrt{(R\;^+_-r_1)(R\;^+_- r_2)}$$
  

tyu.png

 

Chứng minh bổ đề: Ta chứng minh trong trường hợp $C_1$ và $C_2$ tiếp xúc trong với $C$. 

 Ta thấy: $$t_{12}^2=O_1O_2^2-(r_1-r_2)^2$$ Áp dụng định lý cosin, ta có: $$O_1O_2^2=OO_1^2+OO_2^2-2OO_1.OO_2\cos \angle O_1OO_2\\\\AB^2=2r^2(1-\cos \angle O_1OO_2$$ Từ đó thay vào trên ta dễ dàng suy ra: $$t_{12}=\dfrac{AB}{R}\sqrt{(R-r_1)(R- r_2)}$$ Tương tự khi $C_1, C_2$ tiếp xúc ngoài với $C$ thì $$t_{12}=\dfrac{AB}{R}\sqrt{(R+r_1)(R+ r_2)}$$

 

Quay lại bài toán: 

 

46198011.jpg

 

 

Ta kí hiệu $A, B, C, D$ theo thứ tự là tiếp điểm của $C_1, C_2, C_3, C_4$ với $C$, theo bổ đề trên ta có: $$t_{12}t_{34}+t_{14}t_{23}-t_{13}t_{24}=\dfrac{AB.CD+AD.BC-AC.BD}{R^2}.\sqrt{(R-r_1)(R-r_2)(R-r_3)(R-r_4)}$$ Ngoài ra theo định lý Ptolemy, ta có $AB.CD+AD.BC-AC.BD=0$ nên dễ dàng suy ra điều phải chứng minh.

 

 

Lưu ý: Ta chọn các dấu $"+"$ khi tiếp tuyến $t_{ij}$ và $t_{zk}$ không cắt nhau. Trong trường hợp hai tiếp tuyến này cắt nhau ta chọn dấu $''-''$. 

 

Bài toán 1: Cho đường tròn $(O)$ đường kính $AB$. $P, Q$ là hai điểm bất kỳ trên $(O)$ và khác phía với $AB$. Kẻ $QT$ vuông góc với $AB$, $PC, PD$ lần lượt là tiếp tuyến kẻ từ $P$ tới đường tròn đường kính $AT, BT$. Chứng minh rằng: $PC+PD=PQ$

 

Bài toán 2: (IMO 2011) Cho tam giác $ABC$ nội tiếp đường tròn $(O)$. Gọi $l$ là tiếp tuyến bất kỳ của $(O)$. Kí hiệu $l_a, l_b, l_c$ là đường thẳng đối xứng với $l$ qua $BC, CA, AB$ $l_a, l_b, l_c$ cắt nhau tạo thành tam giác $A'B'C'$. Chứng minh rằng đường tròn $(A'B'C') tiếp xúc với $(O)$




#542179 XUNG QUANH ĐẠI LƯỢNG $A-B, B-C, C-A$

Gửi bởi Trung Gauss trong 28-01-2015 - 20:51

XUNG QUANH ĐẠI LƯỢNG $A-B, B-C, C-A$
 

     Nhận xét chung: Khi đứng trước các bài toán bất đẳng thức ở dạng hoán vị hoặc đối xứng, một ý tưởng hiển nhiên là ta sẽ liên tưởng tới nhận xét: "Mọi đa thức đối xứng đều có thể biểu diễn dưới dạng đối xứng cơ sở". Cụ thể là với đa thức hoán vị vòng quanh $f(a,b,c)$ thì ta có: $$f(a,b,c)=\dfrac{1}{2}\left[ f(a,b,c)+f(c,b,a)\right] +\dfrac{1}{2}\left[ f(a, b, c)-f(c, b, a)\right]$$ Khi đó đại lượng $f(a,b,c)-f(c,b,a)$ có thể phân tích thành các đại lượng theo $a-b, b-c, c-a$. Đồng thời, khi vai trò các biến như nhau, ta hoàn toàn có thể giả sử $a=\min\{a,b,c\}$ hoặc $a=\max\{a, b, c\}$ hoặc $a\ge b\ge c$. Từ đó tìm hướng tiếp cận cụ thể cho bài toán.
 

Bài toán 1:(Trần Nam Dũng, VMO 2008) Cho $a, b, c$ là các số thực không âm đôi một phân biệt. Chứng minh rằng: $$\dfrac{1}{(b-c)^2}+\dfrac{1}{(c-a)^2}+\dfrac{1}{(a-b)^2}\ge\dfrac{4}{ab+bc+ca}$$

  Lời giải: Không giảm tính tổng quát, ta có thể giả sử $c=\min\{a,b,c\}$. Ta nhận thấy: $$(a-c)^2+(b-c)^2=(a-b)^2+2(a-c)(b-c)$$ Suy ra: $$\begin{aligned}\dfrac{1}{(b-c)^2}+\dfrac{1}{(c-a)^2}+\dfrac{1}{(a-b)^2}&=\dfrac{1}{(a-b)^2}+\dfrac{(a-c)^2+(b-c)^2}{(a-b)^2(b-c)^2}\\&=\dfrac{1}{(a-b)^2}+\dfrac{(a-b)^2}{(b-c)^2(a-c)^2}+\dfrac{2}{(a-c)(b-c)}\\&\ge\dfrac{2}{(a-c)(b-c)}+\dfrac{2}{(a-c)(b-c)}=\dfrac{4}{(a-c)(b-c)}\end{aligned}$$ Mặt khác, do $c=\min\{a, b, c\}$ nên: $$c(2a+2b-c)\ge 0\Rightarrow \dfrac{4}{(a-c)(b-c)}\ge\dfrac{4}{ab+bc+ca}$$ Từ đó ta có đpcm.
 

Cách khác:

Không mất tính tổng quát, giả sử $a>b>c\ge 0$. Đặt $x=a-b, y=b-c$ Khi đó, $x, y>0$ và: $$ab+bc+ca\ge ab=(c+y)(c+x+y)\ge y(x+y)$$ Theo đó, ta lui về chứng minh một kết quả mạnh hơn: $$\begin{aligned} &\;\;\;\;\;\;\dfrac{1}{x^2}+\dfrac{1}{y^2}+\dfrac{1}{(x+y)^2}\ge\dfrac{4}{y(x+y)}\\&\Leftrightarrow y(x+y)\left[\dfrac{1}{x^2}+\dfrac{1}{y^2}+\dfrac{1}{(x+y)^2}\right]\ge 4\\&\Leftrightarrow \dfrac{y}{x}\left( 1+\dfrac{y}{x}\right)+\dfrac{x}{y}+\dfrac{y}{x+y}\ge 3\end{aligned}$$ Đặt $t=\dfrac{x}{y}$, lúc này ta cần chứng minh: $$\dfrac{t+1}{t^2}+t+\dfrac{1}{t+1}\ge 3$$ Bất đẳng thức này tương đương với: $$\dfrac{(t^2-t-1)^2}{t^2(t+1)}\ge 0$$ Đây là một kết quả hiển nhiên. Bài toán được chứng minh

 

. Bài toán 2: Cho $a, b, c$ là các số thực phân biệt. Chứng minh rằng: $$\left[(a-b)^4+(b-c)^4+(c-a)^4\right]\left[\dfrac{1}{(a-b)^4}+\dfrac{1}{(b-c)^4}+\dfrac{1}{(c-a)^4}\right]\ge\dfrac{297}{8}$$

  Lời giải:

Không giảm tính tổng quát, ta có thể giả sử $a\ge b\ge c$. Đặt $x=a-b, y=b-c$. Khi đó ta có $x, y\ge 0$ và $c-a=-x-y$. Bất đẳng thức cần chứng minh tương đương với: $$\left[ x^4+y^4+(x+y)^4\right]\left[\dfrac{1}{x^4}+\dfrac{1}{y^4}+\dfrac{1}{(x+y)^4}\right]\ge\dfrac{297}{8}$$ Tới đây, ta có đánh giá: $$x^4+y^4\ge\dfrac{(x+y)^4}{8}$$ và $$\dfrac{1}{x^4}+\dfrac{1}{y^4}\ge\dfrac{32}{(x+y)^4}$$ Suy ra: $$\left[ x^4+y^4+(x+y)^4\right]\left[\dfrac{1}{x^4}+\dfrac{1}{y^4}+\dfrac{1}{(x+y)^4}\right]\ge\left[\dfrac{(x+y)^4}{8}+(x+y)^4\right]\left[\dfrac{32}{(x+y)^4}+\dfrac{1}{(x+y)^4}\right]=\dfrac{297}{8}$$ Phép chứng minh hoàn tất.
 

  Bài toán 3: (Trần Quốc Anh) Cho $a, b, c$ là các số thực không âm thỏa mãn $a+b+c=3$. Tìm giá trị lớn nhất của biểu thức: $$P=(a^3+b^3+c^3)(a^2-b^2)(b^2-c^2)(c^2-a^2)$$

  Lời giải:

  Do cần tìm giá trị lớn nhất của $P$ nên ta chỉ cần xét trong trường hợp $a\le b\le c$ là được. Áp dụng bất đẳng thức AM-GM, ta có đánh giá: $$\begin{aligned}(a+b+c)^3&=a^3+b^3+c^3+3(a+b)(b+c)(c+a)\\&\ge 2\sqrt{3(a^3+b^3+c^3)(a+b)(b+c)(c+a)}\end{aligned}$$ Suy ra: $$(a^3+b^3+c^3)(a+b)(b+c)(c+a)\le\dfrac{243}{4}$$ Do $a\le b\le c$ nên $$(a-b)^2\le b^2, (c-a)^2\le c^2$$. Kéo theo: $$\begin{aligned}(a-b)^2(b-c)^2(c-a)^2&\le b^2c^2(b-c)^2=\dfrac{1}{4}.2bc.2bc.(b^2-2bc+c^2)\\&\le \dfrac{1}{4}\left(\dfrac{2bc+2bc+b^2-2bc+c^2}{3}\right)^3=\dfrac{(b+c)^6}{108}\le\dfrac{(a+b+c)^6}{108}=\dfrac{27}{4}\end{aligned}$$ Suy ra: $$(a-b)(b-c)(c-a)\le \dfrac{3\sqrt{3}}{2}$$ Từ đó, ta có ngay: $$P=(a^3+b^3+c^3)(a^2-b^2)(b^2-c^2)(c^2-a^2)\le\dfrac{729\sqrt{3}}{8}$$ Dấu đẳng thức xảy ra khi và chỉ khi $(a,b,c)=(0,3-\sqrt{3},3+\sqrt{3})$ và các hoán vị tương ứng. Kết luận $\max P=\dfrac{729\sqrt{3}}{8}$.
 

  Bài toán 4: (Phan Thành Nam) Cho $a, b, c$ là các số thực không âm. Chứng minh rằng: $$(a^2+b^2+c^2)^2\ge 4(a+b+c)(a-b)(b-c)(c-a)$$

  Lời giải:

  Ta chỉ cần xét trong trường hợp $a\le b\le c$ là đủ. Sử dụng bất đẳng thức AM-GM, ta có: $$\begin{aligned}4(a+b+c)(a-b)(b-c)(c-a)&=4(a+b+c)(b-a).(c-b)(c-a)\\&\le\left[(a+b+c)(b-a)+(c-b)(c-a)\right]^2\\&=\left[b^2+c^2+(b-2c)a-a^2\right]^2\\&\le (b^2+c^2)^2\le (a^2+b^2+c^2)^2\end{aligned}$$ Phép chứng minh hoàn tất.
 

  Bài toán 5: Cho $a, b, c$ là các số thực thỏa $a^2+b^2+c^2=1$. Tìm giá trị lớn nhất và nhỏ nhất của: $$P=(a+b+c)(a-b)(b-c)(c-a)$$

  Lời giải:

  Xét $$P^2=(a+b+c)^2(a-b)^2(b-c)^2(c-a)^2$$ Đặt $p=a+b+c, q=ab+bc+ca, r=abc$. Khi đó ta có: $$(a-b)^2(b-c)^2(c-a)^2=-27r^2+2(9pq-2p^3)r+p^2q^2-4q^3$$ Suy ra: $$P^2=(1+2q)(-27r^2+2(9pq-2p^3)r+p^2q^2-4q^3)$$ Xem $P^2$ như là một tam thức bậc hai ẩn $r$. Ta tính được: $$\Delta '=(1+2q)^2\left[(9pq-2p^3)^2+27(p^2q^2-4q^3)\right]$$ Chú ý rằng $p^2-2q=1$ và sau một vài bước biến đổi cơ bản ta dễ dàng suy ra được: $$\Delta '=4(1+2q)^2(1-q)^3$$ Từ đó suy ra, $$P^2\le\dfrac{4(1+2q)^2(1-q)^3}{27}$$ Thực hiện khảo sát hàm số $$f(q)=\dfrac{4(1+2q)^2(1-q)^3}{27}$$ ta dễ dàng suy ra: $$f(q)=\dfrac{4(1+2q)^2(1-q)^3}{27}\le f\left(-\dfrac{1}{8}\right)=\dfrac{81}{512}$$. Từ đó suy ra: $$P^2\le\dfrac{81}{512}$$ hay $$-\dfrac{9\sqrt{2}}{32}\le P\le \dfrac{9\sqrt{2}}{32}$$ Từ đó suy ra:

  + $\min P=-\dfrac{9\sqrt{2}}{32}$ đạt được khi và chỉ khi $(a, b, c)=\left(\dfrac{3\sqrt{6}+2\sqrt{3}}{12}, \dfrac{\sqrt{3}}{6}, \dfrac{2\sqrt{3}-3\sqrt{6}}{12}\right)$ và các hoán vị tương ứng.

  + $\max P=\dfrac{9\sqrt{2}}{32}$ đạt được khi và chỉ khi $(a, b, c)=\left(\dfrac{2\sqrt{3}-3\sqrt{6}}{12}, \dfrac{\sqrt{3}}{6},\dfrac{3\sqrt{6}+2\sqrt{3}}{12}\right)$ và các hoán vị tương ứng.

 

   Nếu ta gọi $M$ là giá trị lớn nhất (hay nhỏ nhất) của $P$. Sau khi thuần nhất hóa, ta cần định giá trị $M$ nhỏ nhất sao cho bất đẳng thức sau đúng: $(a-b)^2(b-c)^2(c-a)^2(a+b+c)^2\ge M^2(a^2+b^2+c^2)^4$ Dĩ nhiên là lúc này ta bỏ qua điều kiện $a^2+b^2+c^2=1$. Tiếp tục biến đổi ta thu được bài toán của kì thi IMO 2006:

Bài toán:(IMO 2006) Tìm số thực $M$ nhỏ nhất sao cho với mọi $a, b, c$ thực, ta có bất đẳng thức: $$|ab(a^2-b^2)+bc(b^2-c^2)+ca(c^2-a^2)|\le M(a^2+b^2+c^2)^2$$

 

  Bài toán 6: Cho $a, b, c$ là các số không âm có tổng bằng 1. Tìm giá trị lớn nhất của: $$P=(a-b)^3+(b-c)^3+(c-a)^3$$

  Lời giải:

  Ta chú ý thấy $a-b+b-c+c-a=0$ nên dễ dàng suy ra: $$(a-b)^3+(b-c)^3+(c-a)^3=3(a-b)(b-c)(c-a)$$ Theo đó, ta cần tìm giá trị lớn nhất của $$Q=(a-b)(b-c)(c-a)$$ Không mất tính tổng quát, giả sử $a\le b\le c$. Sử dụng bất đẳng thức AM-GM, ta có: $$\begin{aligned}Q&\le\dfrac{1}{2}\left[\dfrac{(\sqrt{3}+1)(b-a)+(\sqrt{3}-1)(c-a)+(c-b)}{3}\right]^3\\&=\dfrac{1}{2}\left(\dfrac{b+c-2a}{\sqrt{3}}\right)^3=\dfrac{1}{2}\left(\dfrac{1-3a}{\sqrt{3}}\right)^3\le\dfrac{1}{2.(\sqrt{3})^3}=\dfrac{\sqrt{3}}{18}\end{aligned}$$ Suy ra: $$P=3Q\le\dfrac{\sqrt{3}}{6}$$ Dấu đẳng thức xảy ra khi $(a,b,c)=\left(0,\dfrac{3-\sqrt{3}}{6}, \dfrac{3+\sqrt{3}}{6}\right)$ và các hoán vị tương ứng. Kết luận: $\max P=\dfrac{\sqrt{3}}{6}$
 

  Bài toán 7: Cho $a, b, c$ là các số thực phân biệt có tổng bằng $1$ và thỏa $ab+bc+ca>0$. Tìm giá trị nhỏ nhất của: $$P=\dfrac{2}{|a-b|}+\dfrac{2}{|b-c|}+\dfrac{2}{|c-a|}+\dfrac{5}{\sqrt{ab+bc+ca}}$$

  Lời giải:

  Không mất tính tổng quát, giả sử $a<b<c$. Khi đó, ta có: $$P=\dfrac{2}{b-a}+\dfrac{2}{c-b}+\dfrac{2}{c-a}+\dfrac{5}{\sqrt{ab+bc+ca}}$$ Áp dụng bất đẳng thức Cauchy Schwarz, ta có: $$P\ge \dfrac{10}{c-a}+\dfrac{5}{\sqrt{ab+bc+ca}}$$ Ngoài ra, theo AM-GM ta còn có: $$\begin{aligned} 3(ab+bc+ca)-1&=3(ab+bc+ca)-(a+b+c)^2\\&=ab+bc+ca-a^2-b^2-c^2\\&=-(c-a)^2+(c-b)(b-a)\\&\le -(c-a)^2+ \dfrac{(c-b+b-a)^2}{4} =-\dfrac{3(c-a)^2}{4}\end{aligned}$$ Suy ra: $$ab+bc+ca\le \dfrac{4-3(c-a)^2}{12}$$ Từ đó suy ra: $$P\ge \dfrac{10}{c-a}+\dfrac{10\sqrt{3}}{\sqrt{4-3(c-a)^2}}$$ Đặt $x=c-a, x>0$ thế thì:$$P\ge \dfrac{10}{x}+\dfrac{10\sqrt{3}}{\sqrt{4-3x^2}}$$ Tới đây, bằng việc khảo sát hàm số $$f(x)=\dfrac{10}{x}+\dfrac{10\sqrt{3}}{\sqrt{4-3x^2}}$$ ta dễ dàng nhận thấy $$f(x)\ge 10\sqrt{6}$$. Cho nên ta có ngay: $$P\ge 10\sqrt{6}$$ Dấu đẳng thức xảy ra khi: $\begin{cases}a+b+c=1\\b-a=c-b\\c-a=\sqrt{\dfrac{2}{3}}\\ab+bc+ca>0\end{cases}\Leftrightarrow \begin{cases}a=\dfrac{1}{3}-\dfrac{1}{\sqrt{6}}\\\\b=\dfrac{1}{3}\\\\c=\dfrac{1}{3}+\dfrac{1}{\sqrt{6}}\end{cases}$

Kết luận, $\min P=10\sqrt{6}$ đạt được khi $(a,b,c)=\left(\dfrac{1}{3}-\dfrac{1}{\sqrt{6}},\dfrac{1}{3},\dfrac{1}{3}+\dfrac{1}{\sqrt{6}}\right)$ và các hoán vị tương ứng.
 

   Cuối cùng, kết thúc bài viết tôi xin đề nghị một số bài tập tự luyện:

   1. (Trần Quốc Anh) Cho $a, b, c$ là các số thực không âm. Chứng minh rằng: $$(a^2+b^2+c^2)^3\ge 27(a-b)^2(b-c)^2(c-a)^2$$

  2. Cho $a, b, c$ là các số thực dương. Chứng minh rằng: $$\sqrt[3]{(a+b)^2(b+c)^2(c+a)^2}\ge\sqrt[3]{(a-b)^2(b-c)^2(c-a)^2}+4\sqrt[3]{a^2b^2c^2}$$

  3. (IMO 1983) Cho $a, b, c$ là độ dài ba cạnh của một tam giác. Chứng minh rằng: $$a^2b(a-b)+b^2c(b-c)+c^2a(c-a)\ge 0$$

  4.(Nguyễn Văn Thạch) Cho các số thực dương $a, b, c$ đôi một phân biệt. Chứng minh rằng: $$\dfrac{1+a^2b^2}{(a-b)^2}+\dfrac{1+b^2c^2}{(b-c)^2}+\dfrac{1+c^2a^2}{(c-a)^2}\ge\dfrac{3}{2}$$

  5. (Võ Quốc Bá Cẩn) Cho $a, b, c$ là các số thực không âm có tổng bình phương bằng $1$. Tìm giá trị lớn nhất của: $$P=(a-b)(b-c)(c-a)(a+b+c)$$

Tài liệu tham khảo: 

$\left[ 1\right]$. Vẻ đẹp bất đẳng thức qua các kỳ thi Olympic - Trần Phương (chủ biên).

$\left[ 2\right]$. Bất đẳng thức hiện đại - Võ Quốc Bá Cẩn.

$\left[ 3\right]$. Một số tài liệu từ Internet.




#539823 Topic ôn luyện VMO 2015

Gửi bởi Trung Gauss trong 06-01-2015 - 11:10

 

 

 Bài 81: Số nguyên lẻ $n\ge 3$ được gọi là "đẹp" khi và chỉ khi tồn tại một hoán vị $(a_1, a_2,..., a_n)$ của các số $(1, 2,..., n)$ sao cho các tổng sao đây đều là các số nguyên dương $$a_1-a_2+a_3-...-a_{n-1}+a_n;\\ a_2-a_3+a_4-...-a_n+a_1;\\a_3-a_4+a_5-...-a_1+a_2;\\...\\a_n-a_1+a_2-...-a_{n-2}+a_{n-1}$$ Hãy xác định tập hợp tất cả các số nguyên dương "đẹp" như vậy.

 

 

 

 

Lời giải:

 

Đặt: $$\begin{cases} y_1=a_1-a_2+a_3-...-a_{n-1}+a_n;\\ y_2=a_2-a_3+a_4-...-a_n+a_1;\\y_3=a_3-a_4+a_5-...-a_1+a_2;\\...\\y_n=a_n-a_1+a_2-...-a_{n-2}+a_{n-1}\end{cases}$$ Lúc này, $n$ sẽ là số "đẹp" khi và chỉ khi tồn tại ít nhất một hoán vị $(a_1, a_2,.., a_n)$ của $(1, 2,..., n)$ sao cho hệ phương trình $$\begin{cases}y_1+y_2=2a_1\;\;\;\;\;\;\;\;\;\;\;\;\;\;\;\;\;\;(1)\\y_2+y_3=2a_3\;\;\;\;\;\;\;\;\;\;\;\;\;\;\;\;\;\;(2)\\...\\y_{n-1}+y_n=2a_{n-1}\;\;\;\;\;\;\;\;\;(n-1)\\y_n+y_1=2a_n\;\;\;\;\;\;\;\;\;\;\;\;\;\;\;\;\;\;(n)\end{cases}$$ có nghiệm nguyên dương.

 

  Cộng vế theo vế các phương trình trong hệ, ta được: $$2(y_1+y_2+...+y_n)=(a_1+a_2+...+a_n)$$ Do $a_1+a_2+...+a_n=1+2+...+n=\dfrac{n(n+1)}{2}$ nên suy ra: $$2(y_1+y_2+...+y_n)=n(n+1)\;\;\;\;\;\;\;\;\;\;\;\;\;\;(*)$$ Cộng từng vế các dòng thứ $i+1, i+3, i+5,...$ cho đến khi quay lại dòng $i+2$, ta có: $$(y_{i+1}+y_{i+2})+(y_{i+3}+y_{i+4})+...+(y_{i-2}+y_{i-1})=2(a_{i+1}+a_{i+3}+...+a_{i-2})$$ Suy ra:  $(y_1+y_2+...+y_n)-y_i=2(a_{i+1}+a_{i+3}+...+a_{i-2})$. Do đó: $$2(y_1+y_2+...+y_n)-2y_i=4(a_{i+1}+a_{i+3}+...+a_{i-2})\;\;\;\;\;\;\;\;\;\;\;\;(**)$$ Từ $(*)$ và $(**)$ suy ra: $$y_i=\dfrac{n(n+1)}{2}-2(a_{i+1}+a_{i+3}+...+a_{i-2})$$ 

 

  Vì $n$ lẻ và $n\ge 3$, nên $n=4k-1$ hoặc $n=4k+1$. Khi đó, ta có

    $\bullet$   Nếu $n=4k-1$ thì $y_i=2k(4k-1)-2(a_{i+1}+a_{i+3}+...+a_{i-2})$ suy ra $y_i$ là số chẵn với mọi $i=\overline{1, n}$.

    $\bullet$   Nếu $n=4k+1$ thì $y_i=(2k+1)(4k+1)-2(a_{i+1}+a_{i+3}+...+a_{i-2})$ suy ra $y_i$ là số lẻ với mọi $i=\overline{1, n}$

 

 Xét hai trường hợp sau:

 

    Trường hợp 1: Nếu $n=4k-1$. Ta sẽ chứng minh rằng với mọi số lẻ $n\ge 3$ thuộc dạng $n=4k-1$ hì $n$ không phải là số "đẹp".

   Thật vậy, giả sử nó là số "đẹp" thì phải tồn tại một hoán vị $(a_1, a_2, ...,a_n)$ của $(1, 2, ..., n)$ sao cho hệ trên có nghiệm. Hơn nữa ta còn phải có $y_i$ là số nguyên dương chẵn với mọi $\overline{1, n}$. Do $(a_1, a_2,..., a_n)$ là một hoán vị của $(1, 2, ..., n)$, nên tồn tại $j (1\le j\le n)$ mà $a_j=1$. Khi đó, xét phương trình thứ $j$, ta có: $$y_j+y_{j+1}=2a_1\Rightarrow y_j+y_{j+1}=2$$ Vì $y_j$ nguyên dương chẵn với mọi $i=\overline{1, n}$ nên $y_j, y_{j+2}\ge 2\Rightarrow y_j+y_{j+2}\ge 4$, mâu thuẫn. Như vậy, với mọi số lẻ $n\ge 3$ thuộc dạng $n=4k-1$ thì $n$ không là số "đẹp"

   Trường hợp 2: Nếu $n=4k+1$,khi đó $y_i$ là số lẻ $\forall i=\overline{1,n}$. Ta chọn hoán vị sau: $$\begin{cases} a_1=2, a_2=4, a_3=6,...,a_{2k}=4k, a_{2k+1}=4k+2\\a_{4k+1}=1, a_{4k}=3, a_{4k-2}=5,...,a_{2k+2}=4k-2\end{cases}$$ Lúc này hệ phương trình ban đầu sẽ nhận các nghiệm: $$\begin{cases}y_1=1, y_2=3, y_3=5,...,y_{2k}=4k-1\\y_{2k+1}=y_{2k+2}=4k+1\\y_{2k+3}=y_{2k+4}=4k-3\\y_{2k+5}=y_{2k+6}=4k-7\\...\\y_{4k-1}=y_{4k}=5\\y_{4k+1}=1\end{cases}$$ Theo đó, số nguyên dương lẻ $n\ge 3$ dạng $n=4k+1$ là số "đẹp".

Cuối cùng, số nguyên dương lẻ $n\ge 3$ là số "đẹp" khi và chỉ khi $n$ có dạng: $$\boxed{n=4k+1,k\in\mathbb{N}}$$




#539472 Topic ôn luyện VMO 2015

Gửi bởi Trung Gauss trong 04-01-2015 - 10:44

Gửi Topic một bài cho vui   :icon6: 

 

 Bài 81: Số nguyên lẻ $n\ge 3$ được gọi là "đẹp" khi và chỉ khi tồn tại một hoán vị $(a_1, a_2,..., a_n)$ của các số $(1, 2,..., n)$ sao cho các tổng sao đây đều là các số nguyên dương $$a_1-a_2+a_3-...-a_{n-1}+a_n;\\ a_2-a_3+a_4-...-a_n+a_1;\\a_3-a_4+a_5-...-a_1+a_2;\\...\\a_n-a_1+a_2-...-a_{n-2}+a_{n-1}$$ Hãy xác định tập hợp tất cả các số nguyên dương "đẹp" như vậy.

 

 

P/S: Trong 2 ngày mà không ai giải bài này thì mình sẽ post lời giải.

Chúc sức khỏe tới các VMFer thi VMO 2015 :)) 




#532537 Thảo luận về DS dự thi và KQ thi VMO 2015

Gửi bởi Trung Gauss trong 09-11-2014 - 15:14

Đội VMO tỉnh Long An:

   1.Châu Hòa Nhân - THPT chuyên Long An

   2. Nguyễn Minh Trí - THPT chuyên Long An 

   3. Võ Minh Khôi - THPT Đức Hòa

   4. Nguyễn Giáp Phương Duy- THPT Hậu Nghĩa

   5. Trần Quốc Khánh - THPT chuyên Long An

   6. Phan Đình Trung - THPT Tân Hưng. 

P/s trong đội có ai là VMFer thì làm quen nhé ^^




#532296 ĐỀ THI HỌC SINH GIỎI LỚP 12 BẢNG B TỈNH LONG AN NĂM HỌC 2014-2015

Gửi bởi Trung Gauss trong 07-11-2014 - 22:13

Nhìn đề em thấy mỗi bài 1 quen quen chắc còn phải cố gắng nhiều  :wub:  :wub: 
C3:
Xét số
$\frac{a}{b^2+c^2}=\frac{a}{3-a^2}$
Ta cần chứng minh
$\frac{a}{3-a^{2}}\leq \frac{a^{2}}{2}$
Thật vậy ta có
$\frac{a}{3-a^2}\leq \frac{a^2}{2}\Leftrightarrow a^4+2a\geq 3a^2$ (đúng theo bđt AM-GM cho 3 bộ $a^4;a;a$)
Tương tự ta có đpcm

 

 Bạn chú ý dấu $\ge$ mới đúng chứ




#531455 ĐỀ THI HỌC SINH GIỎI LỚP 12 BẢNG B TỈNH LONG AN NĂM HỌC 2014-2015

Gửi bởi Trung Gauss trong 02-11-2014 - 07:29

 

KÌ THI CHỌN HỌC SINH GIỎI LỚP 12 CẤP TỈNH VÒNG 2

 Môn: Toán

 

NGÀY I

 

Câu 1 (5,0 điểm)

       Cho ba số dương $a, b, c$ thỏa mãn $a^2+b^2+c^2=3$. Tìm GTNN của: $$P=\dfrac{a}{b^2+c^2}+\dfrac{b}{c^2+a^2}+\dfrac{c}{a^2+b^2}$$

 

 

 

    Cách khác cho bài này. Ta sẽ đi chứng minh bất đẳng thức phụ sau: $$\dfrac{a}{b^2+c^2}\ge\dfrac{1}{2}+\dfrac{1}{2}(a^2-1),\forall a>0$$ Thiết lập hai bdt tương tự từ đó suy ra: $P\ge\dfrac{3}{2}$

 

@ supermember: Bạn Trung Gauss này đừng lên diễn đàn post toàn thứ linh tinh như ở trên nhé :).




#531408 ỨNG DỤNG PHƯƠNG TRÌNH PELL TRONG GIẢI PHƯƠNG TRÌNH NGHIỆM NGUYÊN

Gửi bởi Trung Gauss trong 01-11-2014 - 19:34

SỬ DỤNG PHƯƠNG TRÌNH PELL TRONG GIẢI PHƯƠNG TRÌNH NGHIỆM NGUYÊN

 

 

 

  I. Phương trình Pell:  

      1. Phương trình Pell loại I:

         Phương trình Pell loại I là phương trình nghiệm nguyên có dạng: $$x^2-dy^2=1,\,d\in\mathbb{Z}\;\;\;\;\;\;\;\;\;\;\;(1)$$ 

     Tính chất:   

           1. Nếu $d$ là số chính phương thì phương trình $(1)$ vô nghiệm.

           2. Nếu $d$ là số nguyên âm thì phương trình $(1)$ không có nghiệm nguyên dương.

           3. Phương trình Pell loại I có nghiệm nguyên dương khi và chỉ khi $d$ là số nguyên dương và không chính phương.

      $*$ Công thức nghiệm của phương trình Pell loại I:

           Ta xét trong trường hợp phương trình Pell loại I nhận các nghiệm nguyên dương (tức là $x, y\in\mathbb{Z^+}$). Gọi $(a, b)$ là hai cặp nghiệm bé nhất của phương trình Pell loại I. Khi đó công thức nghiệm của phương trình này được xác định bởi công thức tổng quát của dãy $(x_n), (y_n)$: $$\begin{cases}x_n=\dfrac{(a+b\sqrt{d})^n+(a-b\sqrt{d})^n}{2}\\y_n=\dfrac{(a+b\sqrt{d})^n-(a-b\sqrt{d})^n}{2\sqrt{d}}\end{cases}$$ hay theo công thức truy hồi: $$\begin{cases}x_0=1, y_0=0\\x_1=a, y_1=b\\x_{n+2}=2ax_{n+1}-x_n\\y_{n+2}=2ay_{n+1}-y_n\end{cases}$$

    2. Phương trình Pell loại II:

       Phương trình Pell loại II là phương trình nghiệm nguyên có dạng: $$x^2-dy^2=-1, d\in\mathbb{Z}\;\;\;\;\;\;\;\;\;\;\;\;\;(2)$$

     Tính chất:

          1. Nếu $d$ là số chính phương thì $(2)$ vô nghiệm.

          2. Nếu $d$ có ước nguyên tố dạng $4k+3$ thì $(2)$ vô nghiệm.

          3. Nếu $d$ là số nguyên tố thì $(2)$ có nghiệm khi và chỉ khi $d\neq 4k+3$

          4. (Điều kiện có nghiệm của Phương trình Pell loại II) Gọi $(a, b)$ là nghiệm nhỏ nhất của $(2)$. Khi đó, $(2)$ có nghiệm khi và chỉ khi hệ: $$\begin{cases}a=x^2+dy^2\\b=2xy\end{cases}$$ có nghiệm nguyên dương.

      $*$ Công thức nghiệm của phương trình Pell loại II:

           Xét phương trình Pell loại I liên kết với phương trình $(2)$: $x^2-dy^2=1, d\in\mathbb{Z}$. Gọi $(a, b)$ là nghiệm nhỏ nhất phương trình này. Xét hệ: $$\begin{cases}a=x^2+dy^2\\b=2xy\end{cases}$$  Giả sử hệ này có nghiệm duy nhất và ta gọi $(u, v)$ là nghiệm duy nhất của nó. Khi đó dãy số $(x_n), (y_n)$ sau sẽ vét sạch hết nghiệm của phương trình Pell loại II: $$\begin{cases}x_0=u, y_0=v\\x_1=u^3+duv^2, y_1=dv^3+3u^2v\\x_{n+2}=2ax_{n+1}-y_n\\y_{n+2}=2ay_{n+1}-y_n\end{cases}$$

    3. Phương trình Pell với tham số $n$:

        Phương trình Pell tham số $n$ có dạng: $$x^2-dy^2=n, d,n\in\mathbb{Z}\,\,\,\,\,\,\,\,\,\,\,\,(3)$$

    Tính chất:

         1. Phương trình $(3)$ hoặc vô nghiệm hoặc vô số nghiệm.

         2. Giả sử $(3)$ có nghiệm và gọi $x_0,y_0)$ là nghiệm nguyên dương nhỏ nhất của nó. Gọi $(a, b)$ là nghiệm nguyên dương nhỏ nhất của phương trình Pell loại I tương ứng: $x^2-dy^2=1, d\in\mathbb{Z}$. Khi đó, ta có: $$y_0^2\le\max\left\{nb^2;-\dfrac{na^2}{d}\right\}$$

         3. Giả sử $(3)$ có nghiệm là $(\alpha_1,\beta_1); (\alpha_2, \beta_2);...;(\alpha_m, \beta_m)$ thỏa mãn: $$\beta_i^2\le\max\left\{nb^2,-\dfrac{na^2}{d}\right\}$$ Xét $m$ dãy sau: $$\begin{cases}x_{0,i}=\alpha_i, y_{0,i}=\beta_i\\x_{n+1, i}=ax_{n,i}+day_{n,i}\\y_{n+1,i}=bx_{n,i}+ay_{n,i}\end{cases}$$  Với $(a, b)$ là nghiệm bé nhất của phương trình Pell loại I tương ứng $x^2-dy^2=1$. Đồng thời khi đó các dãy $(x_{n,i}), (y_{n,i})$ sẽ vét hết tất cả các nghiệm của phương trình Pell tham số $n$.

      Phương trình Pell là một công cụ rất mạnh trong việc giải quyết các phương trình nghiệm nguyên cũng như số học nói chung. Ta tìm hiểu các ví dụ sau:

 II. Bài tập ví dụ:

     Bài toán 1: Tìm tất cả các số nguyên dương $x>2$ sao cho tam giác có độ dài ba cạnh là $x-1, x, x+1$ thì diện tích của nó cũng là một số nguyên.

     Lời giải:

         Giả sử $x$ là giá trị thỏa mãn bài toán. Ta có nửa chu vi tam giác đó là: $\dfrac{x+1+x+x-1}{2}=\dfrac{3}{2}x$. Gọi $y$ là diện tích tam giác đó. Áp dụng công thức Heron, ta có: $$y=\sqrt{\dfrac{3}{2}x\left(\dfrac{3}{2}x-x-1\right)\left(\dfrac{3}{2}x-x\right)\left(\dfrac{3}{2}x-x+1\right)}=\dfrac{1}{4}x\sqrt{3(x^2-4)}\\\Leftrightarrow 16y^2=3x^2(x^2-4)$$ Do $16\;\vdots\; 2\Rightarrow x^2(x^2-4)\;\vdots\; 2\Rightarrow x\;\vdots \;2$. Đặt $x=2k,k\in\mathbb{N^*}$. Theo đó: $$6y^2=3,4k^2(4k^2-4)\\\Leftrightarrow y^2=3k^2(k^2-1)\Rightarrow y=k\sqrt{3(k^2-1)}$$ Chú ý rằng: $k,y\in\mathbb{Z^+}\Rightarrow \sqrt{3(k^2-1)}\in\mathbb{Z^+}$. Do đó tồn tại $l\in\mathbb{Z^+}$ sao cho: $$3(k^2-1)=l^2$$ Rõ ràng $l^2\;\vdots \; 3\Rightarrow l=3m,m\in\mathbb{Z^+}$. Khi đó phương trình trở thành $$k^2-3m^2=1$$ Đây chính là phương trình Pell loại I và công thức nghiệm của nó được xác định bởi: $$\begin{cases}k_0=1, m_0=0\\k_1=2, m_1=1\\k_{n+2}=6k_{n+1}-k_n,\forall n=0,1,2,...\\m_{n+2}=6m_{n+1}-m_n,\forall n=0,1,2,..\end{cases}$$ Suy ra: $$\begin{cases}x_0=2,y_0=1\\x_1=4,y_1=2\\x_{n+2}=4x_{n+1}-x_{n},n=0,1,2,...\\y_{n+2}=4y_{n+1}-y_n,n=0,1,2,...\end{cases}$$  Vậy tất cả giá trị của $x$ thỏa mãn bài toán được xác định bởi dãy: $$\begin{cases}x_0=2, x_1=4\\x_{n+2}=4x_{n+1}-x_n. n=0,1,2...\end{cases}$$.

  

    Bài toán 2: Tìm tất cả các số nguyên dương $n$ sao cho trung bình cộng của $n$ số chính phương đầu tiên là một số chính phương.

    Lời giải:

         Bằng qui nạp ta chứng minh được rằng: Với mọi Số nguyên dương $n$, ta luôn có: $$1^2+2^2+3^2+ ...+n^2=\dfrac{n(n+1)(2n+1)}{6}$$ Suy ra: $$\dfrac{1^2+2^2+3^2+...+n^2}{n}=\dfrac{(n+1)(2n+1)}{6}$$ Theo đó, ta cần tìm $n, y\in\mathbb{Z^+}$ sao cho: $$\begin{aligned}&\;\;\;\;\dfrac{(n+1)(2n+1)}{6}=y^2\\&\Leftrightarrow n^2+3n+1=6y^2\\&\Leftrightarrow 16n^2+24n+4=48y^2\\&\Leftrightarrow (4n+3)^2-48y^2=1\end{aligned}$$ Đặt $x=4n+3$, thế thì ta thu được: $$x^2-48y^2=1$$ Đây chính là phương trình Pell loại I. Bằng phép thử tuần tự, ta tìm được $(7,1)$ là nghiệm nhỏ nhất của nó. Theo đó, công thức nghiệm của phương trình Pell này là: $$\begin{cases}x_0=1, y_0=0\\x_1=7, y_1=1\\x_{k+2}=14x_{k+1}-x_k, k=0,1,2,...\\y_{k+2}=14y_{k+1}-y_k,k=0,1,2,...\end{cases}$$ Bằng qui nạp, ta chứng minh được rằng: $$\begin{cases}x_{2k}\equiv 1\pmod{4}\\x_{2k+1}\equiv 3\pmod{4}\end{cases}$$ Ngoài ra: $$\begin{aligned}x_{2k+3}=14x_{2k+2}-x_{2k+1}&=14(14x_{2k+1}-x_{2k})-x_{2k+1}\\&=196x_{2k+1}-14x_{2k}-x_{2k+1}\\&=195x_{2k+1}+x_{2k+1}-14x_{2k}-x_{2k+1}\\&=194x_{2k+1}-x_{2k-1}\end{aligned}$$ Đồng thời, vì $x=4n+3$ nên $n=\dfrac{x-3}{4}\in\mathbb{Z^+}$, theo đó: $n_k=\dfrac{x_{2k+1}-3}{4}\Rightarrow n_0=1, n_1=337, n_{k+1}=194n_k-n_{k-1}+144$. Vậy, tất cả giá trị $n$ thỏa mãn bài toán là: $$\begin{cases}n_0=1, n_1=337\\n_{k+1}=194n_k-n_{k-1}+144, k=0,1,2,...\end{cases}$$                                                      

     Bài toán 3: Tìm cặp số nguyên tố $p, q$ thỏa mãn: $p^2-2q^2=1$

     Lời giải:

          Xét phương trình Pell loại I: $$x^2-2y^2=1$$ Bằng phép thử tuần tự, ta nhận thấy $(3, 2)$ là nghiệm nhỏ nhất của phương trình này đồng thời đây cũng là cặp số nhỏ nhất thỏa mãn bài toán. Theo đó, công thức nghiệm của phương trình Pell này được xác định bởi: $$\begin{cases}x_n=\dfrac{(3+2\sqrt{2})^n+(3-2\sqrt{2})^n}{2}\\y_n=\dfrac{(3+2\sqrt{2})^n-(3-2\sqrt{2})^n}{2\sqrt{2}}\end{cases}$$ Suy ra: $$\begin{aligned}x_n+y_n &=\dfrac{(3+2\sqrt{2})^n+(3-2\sqrt{2})^n}{2}+\dfrac{(3+2\sqrt{2})^n-(3-2\sqrt{2})^n}{2\sqrt{2}}\\&=\dfrac{(\sqrt{2}+1)^{2n+1}+(\sqrt{2}-1)^{2n+1}}{2\sqrt{2}}\end{aligned}$$ Áp dụng công thức khai triển Newton, ta có: $$\begin{aligned}x_n+y_n&=\dfrac{(\sqrt{2}+1)^{2n+1}+(\sqrt{2}-1)^{2n+1}}{2\sqrt{2}}\\&=\dfrac{\sum^{2n+1}_{i=0}C^i_{2n+1}(\sqrt{2})^i+\sum^{2n+1}_{i=0}C^i_{2n+1}(\sqrt{2})^i(-1)^{2n+1-i}}{2\sqrt{2}}\\&=\dfrac{2\sqrt{2}\sum^{n}_{j=0}C^{2j+1}_{2n+1}2^j}{2\sqrt{2}}=\sum^{n}_{j=0}C^{2j+1}_{2n+1}2^j\\&=C^1_{2n+1}+\sum^{n}_{j=1}C^{2j+1}_{2n+1}2^j=1+\sum^n_{j=1}C^{2j+1}_{2n+1}2^j\equiv 1\pmod{2}\end{aligned}$$ Điều này chỉ xảy ra khi $p, q$ khác tính chẵn lẻ hay nói cách khác phương trình đã cho vô nghiệm nếu $\min\{p,q\}>2$. Vậy, $(3, 2)$ là nghiệm duy nhất của bài toán.

   

     Bài toán 4: Tìm tất cả các số tự nhiên $n$ sao cho $2n+1$ và $3n+1$ đều là các số chính phương.

     Lời giải:

         Đặt $d=\gcd (2n+1, 3n+1)$, khi đó: $$\begin{cases}d\;|\;2n+1\\d\;|\;3n+1\end{cases}\Leftrightarrow \begin{cases} d\;|\;6n+3\\d\;|\;6n+2\end{cases}\Rightarrow d\;|\;1\Rightarrow d=1$$ Điều này cho thấy $2n+1, 3n+1$ đồng thời là các số chính phương khi và chỉ khi tồn tại $y\in\mathbb{Z^+}$ sao cho: $$\begin{aligned}&\;\;\;\;(2n+1)(3n+1)=y^2\\&\Leftrightarrow 6n^2+5n+1=y^2\\&\Leftrightarrow 144n^2+120n+24=24y^2\\&\Leftrightarrow (12n+5)^2-24y^2=1\end{aligned}$$ Đặt $x=12n+5$, phương trình trở thành: $$x^2-24y^2=1$$Đây chính là phương trình Pell loại I, vì $24$ không phải là số chính phương nên phương trình này chắc chắn có 

nghiệm và công thức nghiệm của nó được cho bởi: $$\begin{cases} x_0=1, y_0=0\\x_1=5, y_1=1\\x_{k+2}=10x_{k+1}-x_k, k=0,1,2,...\\y_{k+2}=10y_{k+1}-y_k, k=0,1,2,...\end{cases}$$ Bằng qui nạp ta chứng minh được $x_{2k+1}\equiv 5\pmod{12}$, do đó: $$n_k=\dfrac{x_{2k+1}-5}{12}$$ Ngoài ra: $$\begin{aligned}x_{2k+3}&=10x_{2k+2}-x_{2k+1}\\&=10(10x_{2k+1}-x_{2k})-x_{2k+1}\\&=99x_{2k+1}-10x_{2k}\\&=98x_{2k+1}+(10x_{2k}-x_{2k-1})-10x_{2k}\\&=98x_{2k+1}-x_{2k-1}\end{aligned}$$  Suy ra: $n_{k+1}=98n_k-n_{k-1}+40$. Và theo đó tất cả giá trị $n$ thỏa mãn bài toán là: $$\begin{cases}n_0=0, n_1=40\\n_{k+}=98n_k-n_{k-1}+40, k=0,1,2,...\end{cases}$$                                                                          

    Nhận xét: Với cách làm tương tự như bài toán 4, ta có thể giải quyết được bài toán sau:

    Bài toán (Việt Nam TST 2013):

       1. Chứng minh rằng tồn tại vô hạn số nguyên dương $t$ sao cho $2012t+1$ và $2013t+1$ đều là các số chính phương.

       2. Xét $m, n$ là các số nguyên dương sao cho $mn+1$ và $(m+1)n+1$ đều là các số chính phương. Chứng minh rằng $n$ chia hết cho $8(2m+1)$

   

   Bài toán 5: Chứng minh rằng tồn tại vô hạn số nguyên dương $x, y, z$ sao cho: $$x^2+y^3=z^4$$

   Lời giải:

   Bổ đề: Tồn tại vô hạn số nguyên dương $n, k$ sao cho: $$\dfrac{n(n+1)}{2}=k^2$$

Chứng minh bổ đề:

     Biến đổi phương trình $(1)$ về dạng tương đương: $$(2n+1)^2-8k^2=1$$ Đặt $m=2n+1$, phương trình $(1)$ trở thành $m^2-8k^2=1$. Đây chính là phương trình Pell loại I và công thức nghiệm của nó được cho bởi: $$\begin{cases}m_0=1, k_0=0\\m_1=3, k_1=2\\m_{l+2}=6m_{l+1}-m_l\\k_{l+2}=6k_{l+1}-k_l\end{cases}$$ Ngoài ra, ta thấy $m_l\equiv 1\pmod{2}$ nên giá trị $n$ thỏa mãn bổ đề được cho bởi: $$\begin{cases}n_0=0, n_1=1\\n_{l+2}=6n_{l+1}-n_l+2,l=0,1,2,...\end{cases}$$ Dãy số này chứng tỏ khẳng định của bổ đề là đúng. Bổ đề được chứng minh.

 Trở lại bài toán, Bằng qui nạp ta dễ dàng kiểm tra được rằng với mọi $n$ nguyên dương, ta luôn có: $$\begin{aligned}&\;\;\;\;1^3+2^3+3^3+...+n^3=\left[\dfrac{n(n+1)}{2}\right]^2\\&\Leftrightarrow [1^3+2^3+3^3+...+(n-1)^3]+n^3=\left[\dfrac{n(n+1)}{2}\right]^2\\&\Leftrightarrow \left[\dfrac{(n-1)n}{2}\right]^2+n^3=\left[\dfrac{n(n+1)}{2}\right]^2\end{aligned}$$ Từ đây, áp dụng bổ đề trên, ta chọn $$\begin{cases}x=\dfrac{n(n+1)}{2}\\y=n\\z^2=\dfrac{n(n+1)}{2}\end{cases}$$ Rõ ràng đây chính là ba bộ số thỏa mãn bài toán. Từ đó suy ra đpcm.

 

   Bài toán 6: Chứng minh rằng tồn tại vô hạn số nguyên dương $x, y$ sao cho: $$\dfrac{x+1}{y}+\dfrac{y+3}{x}=6$$

   

    Lời giải:

        Phương trình đã cho tương đương với: $$x^2+(1-6y)x+y(y+3)=0$$ Xem đây như một phương trình bậc hai ẩn $x$ tham số $y$, ta có: $$\Delta =(1-6y)^2-4y(y+3)=32y^2-24y+1$$ Bây giờ ta nhận thấy rằng phương trình đã cho có vô hạn nghiệm nguyên dương khi và chỉ khi tồn tại vô hạn số nguyên dương $y,k$ thỏa mãn: $$\begin{aligned}&\;\;\;32y^2-24y+1=k^2\\&\Leftrightarrow 64y^2-48y+2=2k^2\\&\Leftrightarrow (8y-3)^2-2k^2=7\end{aligned}$$ Đặt $m=8y-3$, phương trình trở thành: $$m^2-2k^2=7\;\;\;\;\;\;\;\;\;\;\;\;\;\;\;\;\;\;\;\;\;\;\;\;\;(*)$$ Xét phương trình Pell loại I: $m^2-2k^2=1$, dễ thấy phương trình Pell này nhận nghiệm nhỏ nhất là $(3, 2)$, bằng phép thử tuần tự ta tìm được $(3, 1)$ là nghiệm cơ sở của $(*)$. Theo đó, ta xét dãy $(x_n), (y_n)$: $$\begin{cases}x_0=3, y_0=2\\x_{n+1}=3x_n+4y_n, n=1,2,3,...\\y_{n+1}=2x_n+3y_n,n=1,2,3,...\end{cases}$$ Đồng thời: $$x_{n+1}^2-2y_{n+1}^2=(3x_n+4y_n)^2-2(2x_n+3y_n)^2=x_n^2-2y_n^2=...=x_1^2-2y_1^2=7$$ Điều này chứng tỏ phương trình $m^2-2k^2=7$ có vô hạn nghiệm nguyên dương. Ngoài ra ta còn nhận thấy $x_{2k}\equiv 3\pmod{8}$, thật vậy chú ý rằng: $$\begin{aligned}x_{n+1}&=3x_n+4y_n\\&=9x_{n-1}+12y_{n-1}+4y_n\\&=9x_{n-1}+8y_{n-1}+4(y_n+y_{n-1})\\&=9x_{n-1}+8y_{n-1}+8(x_{n-1}+2x_{n-1})\equiv x_{n-1}\pmod{8}\end{aligned}$$ Như vậy, $$ x_{2k}\equiv x_{2k-2}\equiv ...\equiv x_2\equiv x_0\equiv 3\pmod{8}$$ Điều này cho thấy tồn tại vô hạn $y,k\in\mathbb{Z^+}$ sao cho: $$(8y-3)^2-2k^2=7$$ Từ đó dễ dàng suy ra đpcm.


  Tài liệu tham khảo:

     [1] Phương trình nghiệm nguyên - Phan Huy Khải.

     [2] Phương trình Diophant - Trần Nam Dũng

     [3] Lời giải và bình luận TST 2013.




#531392 ĐỀ THI HỌC SINH GIỎI LỚP 12 BẢNG B TỈNH LONG AN NĂM HỌC 2014-2015

Gửi bởi Trung Gauss trong 01-11-2014 - 18:34

KÌ THI CHỌN HỌC SINH GIỎI LỚP 12 CẤP TỈNH VÒNG 2

 Môn: Toán

 

NGÀY I

 

Câu 1 (5,0 điểm)

       Cho ba số dương $a, b, c$ thỏa mãn $a^2+b^2+c^2=3$. Tìm GTNN của: $$P=\dfrac{a}{b^2+c^2}+\dfrac{b}{c^2+a^2}+\dfrac{c}{a^2+b^2}$$

 

Câu 2 (5,0 điểm)

       Tìm số hạng tổng quát của dãy $(x_n)$ biết rằng: $$\begin{cases}x_0=1, x_1=5, x_2=125\\x_{n+2}x_nx_{n-1}=3(x_{n+1})^2x_{n-1}+10x_{n+1}(x_n)^2\end{cases}(n\in\mathbb{N^*})$$

 

Câu 3 (5,0 điểm)

      Cho tam giác $ABC$ có ba góc nhọn nội tiếp đường tròn tâm $O$. Gọi $M, N, P$ lần lượt là trung điểm $BC, CA, AB$. Gọị $d_1$ là đường thẳng qua $M$ và song song với $OA$, $d_2$ là đường thẳng qua $N$ và song song với $OB$, $d_3$ là đường thẳng qua $P$ và song song với $OC$. Chứng minh rằng $d_1, d_2, d_3$ đồng qui.

 

Câu 4 (5, 0 điểm)

      Viết tất cả các số $\dfrac{1}{2014}, \dfrac{2}{2014},...,\dfrac{2014}{2014}$ lên bảng. Ta thực hiện công việc xóa đi hai số $a, b$ bất kỳ trên bảng đồng thời điền lên bảng một số mới là $a+b-2014ab$. Sau một số hữu hạn lần thực hiện, trên bảng chỉ còn một số. Số đó là số nào?

 

NGÀY II

 

Câu 1 (7,0 điểm)

     Tìm tất cả hàm số $f: \mathbb{R}\to \mathbb{R}$, biết rằng $f$ là hàm chẵn và thỏa mãn: $$f(xy)-f(x)f(y)=2014(f(x+y)-2xy-1),\forall x, y\in \mathbb{R}$$

 

Câu 2 (6,0 điểm)

     Cho $a, b, c$ là các số nguyên thỏa mãn $a^4+b^4+c^4$ chia hết cho $9$. Chứng minh rằng có ít nhất một trong các số $a^2-b^2, b^2-c^2, c^2-a^2$ chia hết cho $9$.

 

Câu 3 (7,0 điểm)

     Cho viên gạch kích thước $1\times 4$ (hình A) và sàn nhà kích thước $10\times 10$ đã bị mất bốn ô vuông (hình B):

 

h%E1%BB%B9.png

 

           (hình A)

 

hy1.png

                                          (hình B)

 

Chứng minh rằng không thể lát 24 viên gạch hình A thành sàn nhà như hình B được.

 

 

P/S: Đề ngày I chém ngon, đề ngày II làm chắc mỗi câu pt hàm, câu 2 làm dư trường hợp, câu 3 lập luận không chặt :(  :(  :(  :( 




#529524 Đề thi chọn Đội tuyển trường THPT chuyên Lê Quý Đôn tỉnh Ninh Thuận

Gửi bởi Trung Gauss trong 19-10-2014 - 11:13

 

 
 Câu 3:
 Cho $a$ là một số nguyên khác $0$. Chứng minh rằng mọi ước nguyên tố của số: $a^{2.6^{n}}-a^{6^{n}}+1$ đều có dạng $6^{n+1}.k+1$, với $n, k$ là các số nguyên dương.
 
 

   Gọi $p$ là một ước nguyên tố của $a^{2.6^{n}}-a^{6^n}+1$. Ta có: $$a^{2.6^n}-a^{6^n}+1\equiv 0\pmod{p}\\\Rightarrow a^{6^n}(1-a^{6^n})\equiv 1\pmod{p}\\\Rightarrow -a^{2.6^n}.a^{6^n}\equiv 1\pmod{p}\Rightarrow a^{6^{n+1}}\equiv 1\pmod{p}\Rightarrow \text{ord}_p(a)\;|\;6^{n+1}$$ Điều này chỉ xảy ra khi: $\text{ord} _p(a)\in \left\{ 2^k, 3^k, 6^k\right\}, k=\overline{1, n+1}$

  Nếu $\text{ord}_p(a)\in\left\{2^k, 3^k, 6^m\right\}, \forall k=\overline{1,n+1},\forall m=\overline{1,n}$ thì: $$a^{2.6^n}-a^{6^n}+1\equiv 1\pmod{p}, \text{mâu thuẫn}$$ Vậy, $\text{ord}_p(a)=6^{n+1}$. Áp dụng định lý Fermat, ta có: $$\text{ord}_p(a)\;|\;p-1\Rightarrow 6^{n+1}\;|\;p-1\Rightarrow p=6^{n+1}.k+1,\forall k\in \mathbb{N^*}$$